You are on page 1of 47

OPTICS

DHANALAKSHMI NAGAR
NEAR ANNAMAIAH CIRCLE,
TIRUPATI.
PH NO. 9440025125

OPTICS

Review of Concepts
(a)
(B)

Due to reflection, none of frequency, wavelength and speed of light change.


Law of reflection :
(i) Incident ray, reflected ray and normal on inident point are coplanar.
(ii) The angle of incidence is equal to angle of reflection.

Some important points : In case of plane mirror :


(i) For real object, image is virtual.
(ii) For virtual object, image is real.
(iii) Image size = Object size.
(iv) The converging point of incident beam behaves as object.
(v) If incident beam on optical instrument (mirror, lens etc) is converging in nature, object is
virtual.
(vi) If incident beam on the optical instrument is diverging in nature, the object is real.
(vii) The converging point of reflected or refracted beam from an optical instrument behaves a
image.
(viii) If reflected beam or refracted beam from an optical instrument is converging in nature,
image is real.

n
P

Real
Image

Virtual
Object

(ix) If reflected beam or refracted beam from


an optical instrument is diverging in nature,
image is virtual.

P
n
Real
Object

Virtual
Object

(x) For solving the problem, the reference frame is chosen in which optical instrument (mirror,
lens, etc.) is in rest.
(xi) The formation of image and size of image is independent of size of mirror.

(xii) Visual region and intensity of image


depend on size of mirror.

www.physicsashok.in

OPTICS
(xiii) If the plane mirror is rotated through an angle , the reflected ray and image is rotated
through an angle 2 in the same sense.
(xiv) If mirror is cut into a number of pieces, then the focal length does not change.
(xv) The minimum height of mirror required to see the full image of a man of height h is h/2.
v sin

Rest

Rest

v sin

Object

Image

Object

v cos Image

v cos

(xvi)

(xvii)

Rest
vm

vm
Object

Image

2 vm - v

vm

2 vm

(xix)

(xviii)

Object

Image

Object
In rest

Image

2 vm + v

(xx)
(C) Number of images formed by combination of two plane mirrors : The images formed by
combination of two plane mirrors are lying on a circle whose centre is at the meeting points of mirrors.
Also, object is lying on that circle.
360

where = angle between mirrors.

Here, n

(i) If

360
is even number, the number of image is n1.

360
is odd number and object is placed on bisector of angle between mirrors, then

number of images is n1.

(ii) If

360
is odd and object is not situated on bisector of angle between mirrors, then the

number of images is euqal to n.

(iii) If

(D)

Law of reflecteion in vector form :


Let e1 = unit vecotr along incident ray..

Let e 2 = unit vector along reflected ray

n = unit vector along normal on point of incidence


Then,
e 2 e1 2 e1 . n n

(e)

Spherical mirrors :
(i) It is easy to solve the problems in geometrical optics by the help of co-ordinate sign convention.

www.physicsashok.in

OPTICS
y

x x

x
y

x x
y

(ii) The mirror formula is

x x
y

x x

1 1 1

u

Also, R = 2
These formulae are only aplicable for paraxial rays.
(iii) All distances are measured from optical centre. It means optical centre is taken as origin.
(iv) The sign convention are only applicable in given values.
(v) The transverse magnification is
image size

object size
u
Sun
1. If object and image both are real, is negative.
2. If object and image both are virtual, is negative
D
F

3. If object is real but image is virtual; is positive.

4. If object is virtual but image is real, is positive.


d
5. Image of star; moon or distant object is formed
at focus of mirror.
If
y = the ddistance of sun or moon from earth.
D = diameter of moon or suns disc.
= focal length of the mirror
d = diameter of the image
= the angle subtended by sun or moons disc
D d

Then tan = =
y
Here, is in radian.

Laws of Refraction
1.

(a)The incident ray, the refracted ray and normal on incidence point are coplanar.

1
1
(B) 1 sin 1 2 sin 2 cons tan t .

2
2

n
1

(C) Snells law in vector form:


Let,

e1 = unit vector along incident ray

e 2 = unit vecotr along refracted.

www.physicsashok.in

OPTICS
n = unit vector along normal on incidence point.
Then 1 e1 n 2 e 2 n .

Some important points :


(i) The value of absolute retractive index is always greater or equal to one.
(ii) The value of refractive index depends upon material of medium, colour of light and temperature
of medium.
(iii) When temperature increases, refractive index decreases.
(iv) Optical path is defined as product of geometrical path and refractive index.
i.e.,
optical path = x
(v) For a given time, optical path remains constant.
i.e.,

1 x1 2 x 2 cons tan t

1 c1 2 c 2
2 c1

1 c 2

i.e.,

dx 1
dx 2
2
dt
dt

1
c

(vi) The frequency of light does not depend upon medium.


c1 1 ,
c2 2

1 c 2 2
1

2
c1
1

(a) When observer is in rarer medium and object is in denser medium:


real depth

Then
apparent depth
(B) When object is in air and observer is in
denser medium:
Apparent
Real
depth P
apparent position
depth

real position

2.

Air
Observer

Denser medium
()
P

Object

1
(C) The shift of object due to slab is x t 1

(i) This formula is ony applicable when observer is in


rarer medium.
(ii) The object shiftness does not depend upon the
position of object.
(iii) Object shiftness takes place in the direction of
incidence ray.

Object
shiftness
=x

(D) The equivalent rerfractive index of a combination of a number of slabs for normal incidence

is

ti
t
i
i

Here,

t i = t 1 + t 2 + .......

1
2

t1
t2

ti
t
t
1 2
i 1 2

www.physicsashok.in

OPTICS
(e) The apparent depth due to a number of media is

ti
.
i

(f) The lateral shifting due to a slab is d = t sec r sin (i r).

t
d

3.

Rarer
1

(a) Cricital angle : When a ray passes from denser medium ( 2 )


90

to rarer medium ( 1 ), then for 90 angle of refraction, the


corresponding angle of incidence is critical angle.
Mathematically,

Rarer medium
(1)

(B) (i) When angle of incidence is lesser than


critical angle, refraction takes place. the
corresponding deviation is

sin 1 2 sin i i
for i = C

(ii) When angle of incidence is greater than


critical angle, total internal reflection takes
place. the corresponding deviation is

2i

4.

Denser
2

sin C 1
2

when i > C

i<C

i=C

Denser medium
(2)

The i graph is
(i) Critical angle depends upon colour of light, material of medium,
and temperature of medium.
(ii) Critical angle does not depend upon angle of incidenct.

/2

PRISM
A
(a) Deviation produced by prism is i i A .
(B) r + r = A
(C) For grazing incidence, i = 90
(D) For grazing emergence, i = 90
A
(e) For not transmitting the ray from prism, > cosec
2
(f) For limiting angle of prism, i = i = 90, the limiting
angle of prism = 2C where C is critical angle. If angle
of prism exceeds the limiting values, then the rays
are totally reflected.
(g) i graph for prism:
(h) For minimum deviation,

(i) i = i and r = r

A
sin m

(ii)
A
sin
2

i
r

m
i

www.physicsashok.in

OPTICS
In the case of minimum deviation, ray is passing through prism symmetrically.
(i) For maximum deviation max .
i = 90 or i = 90
(j) For thin prism, 1 A
(k) Angular dispersion, D r A

(l) Angular deviation, y y 1 A


r

(m) dispersive power = 1


y

(n) y
2

(o) For dispersion without deviation, y 0


(p) For deviation without dispersion, D 0
Refractive surface formula,

2 1 2 1

u
r
Here, = image distance, u = object distance, r = radius of curvature of spherical surface
(a) For plane surface, r =
(B) Transverse magnification,

Im age size 1

object size 2u

(C) Refractive surface formula is only applicable for paraxial ray.

LENS
1.

1 1 1

u

Lens formula :

(a) Lens formula is only applicable for thin lens.


(B) r = 2 formula is not applicable for lens.
(C) m

image size

object size u

(D) Magnification formula is only applicable when object is perpendicualr to optical axis.
(e) Lens formula and the magnification formula is only applicable when medium on both sides of
lenses are same.

f(+ve)

(f)

f(+ve)
(i)

f(-ve)
(ii)

f(-ve)

f(-ve)
(iii)

(iv)

f(+ve)

(v)

(g) This lens formula is applicable for converging as well diverging lens.
Thin lens makers formula :

(vi)
1

1 2 1 1 1

1 r1 r2

www.physicsashok.in

OPTICS
2.

(a) Thin lens formula is only applicable for paraxial ray.


(B) This formula is only applicable when medium on both sides of lens are same.
(C) Intensity is proportional to square of aperture.
(D) When lens is placed in a medium whose refractive index is greater than that of lens. i.e.,

1 2 . Then converging lens behaves as diverging lens and vice versa.


(e) When medium on both sides of lens are not same. Then both focal lengths are not same to
each other.
(f) If a lens is cut along the diameter, focal length does not change.

(g) If lens is cut by a vertical, it converts into two lenses of different


focal lengths.

1
1
1

1 2

i.e.,

f1
f

f2

1
2
3
+ + +
+ + + 4

(h) If a lens is made of a number of layers of different refractive


index (shown in figure).
Then number of images of an object formed by the lens is
equal to number of different media.

5
6

(i) The minimum distance between real object and image in is 4 .

f1

(j) The equivalent focal elngth of co-axial combination of two


lenses is given by
1
1
1
d

F 1 2 1 2

f2
d<f1

o1

d<f2
o2

(k) If a number of lenses are in contact, then

1
1
1

F 1 2

(i) Power of thin lens, P

(l)

1
F

1
F
(m) If a lens is silvered at one surface, then the system behaves as an equivalent mirror, whose
power P = 2PL + Pm

(ii) Power of mirror is P

Here,

2 1 1 1

PL = Power of lens =
r r
1

1 2
1
Pm = Power of silvered surface F

r2
, where r2 = radius of silvered surface.
2

Here,

Fm

Here,

1
F
F = focal length of equivalent mirror.
P

www.physicsashok.in

OPTICS

ASSERTION & REASION


THE NEXT QUESTIONS REFER TO THE FOLLOWING INSTRUCTIONS
A statement of assertion (A) is given and a Corresponding statement of reason (R) is given just
below it of the statements, mark the correct answer as
(A) If both A and R are true and R is the correct explanation of A.
(B) If both A and R are true but R is not the correct explanation of A.
(C) If A is true but R is false.
(D) If both A and R are false.
(E) If A is false but R is true.
1.

Assertion (A) :
Reason (R) :
(A)
(B)

A single ray cant be isolated from a source however small it may be.
The concept of single ray is hypothetical.
(C)
(D)
(E)

2.

Assertion (A) :
Reason (R) :
(A)
(B)

Virtual images can be photographed.


Rays from virtual images are diverging.
(C)
(D)
(E)

3.

Assertion (A) :
Reason (R) :
(A)
(B)

Virtual object cant be seen by human eye.


Virtual object is formed by converging rays.
(C)
(D)
(E)

4.

Assertion (A) :
Reason (R) :
(A)
(B)

A Convex mirror is used as rear view mirror.


The Convex mirror always forms virtual, erect and diminished image.
(C)
(D)
(E)

5.

Assertion (A) :
Reason (R) :
(A)
(B)

The behavior of any lens depends on surrounding medium.


A lens can be looked upon as a collection of small prism with varying prism angle.
(C)
(D)
(E)

6.

Assertion (A) :
Reason (R) :
(A)
(B)

Human eye can see virtual object.


Virtual object is formed by apparent intersection of incident rays.
(C)
(D)
(E)

7.

Assertion (A) :
Reason (R) :
(A)
(B)

Real image is formed by real intersection of reflected or refracted rays.


Real image cant be obtained on screen.
(C)
(D)
(E)

8.

Assertion (A) :
Reason (R) :

If a portion of lens or mirror is blocked or removed, then intensity of image reduces.


As every portion of lens or mirror forms image, hence blocking or removing a portion
will result in intensity reduction.
(C)
(D)
(E)

(A)
9.

(B)

Assertion (A) :
Reason (R) :
(A)
(B)

10. Assertion (A) :

A rectangular glass slab produces no deviation and no dispersion.


Dispersive power of glass is zero.
(C)
(D)
(E)
A double convex lens

1.5

has focal length 10 cm. When immersed in water

, its focal length becomes 40 cm.


3

Reason (R) :
(A)

(B)

1 l m 1 1


f
m R1 R2
(C)

(D)

(E)

www.physicsashok.in

OPTICS
11. Assertion (A) :

A convex lens of glass

1.5 behave as diverging lens when immersed in carbon

disulphide of higher refractive index


Reason (R) :
(A)
(B)
12. Assertion (A) :
Reason (R) :
(A)
(B)
13. Assertion (A) :
Reason (R) :
(A)
(B)
14. Assertion (A) :

1.65 .

A diverging lens is thinner in the middle and thicker at the edges.


(C)
(D)
(E)
A biconvex lens of focal length 10 cm is split into two equal parts by a plane parallel to
its principal axis. The focal length of each part will be 20 cm.
The focal length depends on how many parts the convex lens has been split.
(C)
(D)
(E)
Radius of curvature of a convex mirror is 20 cm. If a real object is placed at 10 cm
from pole of the mirror, image is formed at infinity.
When object is placed at focus, its image is formed at infinity.
(C)
(D)
(E)
For a prism of refracting angle 60 and refractive index

2 , minimum deviation is

30.
Reason (R) :

At minimum deviation, r1 r2

(A)

(C)

(B)

(D)

A
30
2

(E)

15. Assertion (A) :


Reason (R) :
(A)
(B)

Image formed by concave lens is not always virtual.


Image formed by a lens if the image is formed in the direction of ray of light with
(C)
(D)
(E)

16. Assertion (A) :

Minimum deviation for a given prism does not depend on the refractive index
the prism.

Reason (R) :

Deviation by a prism is given by

(A)

(C)

(B)

(D)

of

i1 i2 A and does not have the term .

(E)

Level # 1.
Objective Type Question
Multiple Choice Question with ONE correct answer :
1.

Two plane mirrors M1 and M2 are inclined to each other at 70. A ray incident on the mirror M1 at an angle
falls on M2 and is then reflected parallel to M1 for
(A) = 45
(B) = 50
(C) = 55
(D) = 60

2.

A light ray is incident on a horizontal plane mirror at an angle of 45.


At what angle should a second plane mirror be placed in order that the
reflected ray finally be reflected horizontally from the second mirror,
as shown in figure.
(A) = 30
(B) = 24
(C) = 22.5
(D) = 67.5

3.

A plane mirror is placed in y-z plane facing towards negative x-axis. The mirror is moving parallel to yaxis with a speed of 5 cm/s. A point object P is moving infront of the mirror with a velocity (3 cm/s) i + (4
cm/s) j + (5 cm/s) k . Find the velocity of image with respect to mirror
(A) (3 cm/s) i + (4 cm/s) j + (5 cm/s) k

(B) (3 cm/s) i + (4 cm/s) j + (5 cm/s) k

(C) (3 cm/s) i (4 cm/s) j (5 cm/s) k

(D) none of the above.

www.physicsashok.in

OPTICS
4.

The size of the face of a dancer is 24 cm x 16 cm. Find the minimum size of a plane mirror required to see
the face of dancer completely by
(i) one eyed dancer.
(ii) two eyed dancer.
(Distance between the eyes is 4 cm.)
(A) (i) 12 x 8 cm 2
(ii) 12 x 6 cm 2
2
(B) (i) 8 x 10 cm
(ii) 12 x 2 cm 2
2
(C) (i) 10 x 12 cm
(ii) 9 x 8 cm 2
2
(D) (i) 12 x 2 cm
(ii) 6 x 13 cm 2

5.

A bullet of mass m 2 is fired from a gun of mass m 1 with horizontal velocity v. A plane mirror is fixed at gun
facing towards bullet. The velocity of the image of bullet formed by the plane mirror with respect to bullet
is
m2
m1 m 2
2 m1 m 2

(A) 1
(B)
(C)
(D) none of these
m1
m
m
1
1

6.

In the given figure, the angle of reflection is


(A) 30
(B) 60
(C) 45
(D) none of these.

7.

Two plane mirrors A and B are aligned parallel to each other, as shown in
figure. A light ray is incident at an angle of 30 at a point just inside one
end of A. The plane of incidence coincides with the plane of figure. The
maximum number of times the ray undergoes reflections (excluding the
first one) before it emerges out is
(A) 28
(B) 30
(C) 32
(D) 34

8.

9.

A
2 3m
B

A point source of light B is placed at a distance L in front of the centre of a


d
mirror of width d hung vertically on a wall as shown. A man walks in front of
the mirror along a line parallel to the mirror at a distance 2L from it as shown.
The greatest distance over which he can see the image of the light source
in the mirror is
(A) d
(B) d
(C) 2d
(D) 3d
A plane mirror having a mass m is tied to the free end of a massless
spring of spring constant k. The other end of the spring is attached to
a wall. The spring with the mirror held vertically to the floor can slide
along it smoothly. When the spring is at its natural length, the mirror
is found to be moving at a speed of v cm/s. The separation between
the images of a man standing before the mirror, when the mirror is
in its extreme positions

30

0.2 m

L
2L

k
Wall

m
m
v m
m
(B)
(C) 2 v
(D) 4 v
2 k
k
k
k
10. Two spherical mirrors M1 and M2, one convex and other concave having same radius
of curvature R are arranged coaxially at a distance 2R (consider their pole separation
to be 2R). A bead of radius a is placed at the pole of the convex mirror a shown. The
ratio of the size of the first three images of the bead is
1 1
1 1 1
:
(A) 1 : 2 : 3
(B) 1 : :
(C) :
(D) 3 : 11 : 41
2 3
3 11 41
M
(A) v

M2

11. An object is placed in front of a convex mirror at a distance of 50 cm. A plane mirror is introduced
covering the lower half of the convex mirror. If the distance between the object and the plane mirror is 30
cm, there is no parallax between the images formed by the two mirrors. The radius of curvature of the
convex mirror (in cm) is
(A) 60
(B) 50
(C) 30
(D) 25

www.physicsashok.in

10

OPTICS
12. A rectangular glass slab ABCD of refractive index n1, is immersed in water of refractive index n2 (n1 > n2).
A ray of light is incident at the surface AB of the slab as shown. The maximum value of the angle of
incidence max , such that the ray comes out only from the other surface CD is given by

1 1
1
(B) sin n1 cos sin n

1 n 2
1 n1
(A) sin n cos sin n
2
1

1 n1
1 n 2
(C) sin
(D) sin
n1
n2
13. Two thin slabs of refractive indices 1 and 2 are placed parallel to
each other in the x-z plane. If the direction of propagation of a ray

in the two media are along the vectors r


then we have
(A) 1a 2b

(B)

(C) 1 (a2 + b2) = 2 (c2 + d2)

a i b j and r

1a

y
2

c i d j

2c

a2 b2
c 2 d2
(D) none of these

14. A man stands on a glass slab of height and inside an elevator accelerated upwards with a. The
bottom of the slab appears to have shifted with respect to the man by a distance (if the R. I. of the glass
is g )
(A) less then g

(B) greater than g

(C) equal to g

(D) cant be said.

15. A ray of light travels from a medium of refractive index into air. If the angle of incidence at the plane
surface of separation is and the corresponding angle of deviation is D, the variation D with is shown
correctly by the figure.

D
D1

(0, 0)

(0, 0)

/2

(A)

D1

D2
C

(0, 0)

(B)

D1

D2
C

(0, 0)

/2

(C)

3
2

(B)

3
2

(C)

5
2

(D)

5
.
2

/2

(D)

16. An observer can see through a pin hole at the top end of a thin rod of height
h placed as shown in the figure. Beaker height is 3h and its radius is h.
When the beaker is filled with a liquid up to a height 2h, he can see the
3h
lower end of the rod. Then refractive index of the liquid is
(A)

D2

Eye

h
2h

(Assume that the distance between rod and the wall is negligible).
17. A glass sphere of radius 5 x 102 m has a small bubble 2 x 102 m from its centre. Bubble is viewed along
the diameter of the sphere, from the side on which it lies. If refractive index of glass is 1.5 then how far
from the surface will the bubble appear?
(A) 2.1 cm
(B) 2.5 cm
(C) 1.5 cm
(D) 2.0 cm

www.physicsashok.in

11

OPTICS
18. A ray of light travelling in a transparent medium falls on a surface separating the medium from air at an
angle of incident 45. The ray undergoes total internal reflection. If is the refractive index of the medium
with respect to air, select the possible value(s) of from the following :
(A) 1.3
(B) 1.4
(C) 1.5
(D) 1.7
P

19. A tank contains a transparent liquid of refractive index n the bottom of which
is made of a mirror as shown. An object O lies at a height d above the mirror.
A person P vertically above the object sees O and its image in the mirror and
finds the apparent separation to be
(A) 2nd

(B)

2d
n 1

(C)

2d
n

(D)

O
d

d
1 n
n

20. A fish looks up at the surface of a perfectly smooth lake. The surface appears dark except a circular
area directly above it. The plane angle that this illuminated region subtends is
(A) 48.6
(B) 24.3
(C) 97.2
(D) 12.15
21. A ray of light enters an anisotropic medium from vacuum at grazing incidence. If is the angle made by
the reflected ray inside the medium with the interface and n( ) is the refractive index of the medium
then,
n()
n()
1
1
(A) n( ) sin = 1
(B) n( ) cos = 1
(C)
(D)
sin
cos
22. The slab of a material of refractive index 2 shown in figure has
a curved surface. APB of radius of curvature 10 cm and a plane
surface CD. On the left of APB is air and on the right of CD is
water with refractive indices as given in figure. An object O is
placed at a distance of 15 cm from pole P as shown. The
distance of the final image of O from P, as viewed from the left is
(A) 20 cm
(B) 30 cm
(C) 40 cm
(D) 50 cm
23. An object is placed at a distance of 12 cm from a convex lens on its principal axis and a virtual image of
certain size is formed. On moving the object 8 cm away from the lens, a real image of the same size as
that of virtual image is formed. The focal length of the lens in cm is
(A) 15
(B) 16
(C) 17
(D) 18
24. A spherical surface of radius of curvature R separates air (refractive index 1.0) from glass (refractive
index 1.5). The centre of curvature is in the glass. A point object P placed in air is found to have a real
image Q in the glass. The line PQ cuts the surface at the point O and PO = OQ. The distance PO is equal
to
(A) 5 R
(B) 3 R
(C) 2 R
(D) 1.5 R
25. A lens of focal length is placed in between an object and screen fixed at a distance D. The lens forms
two real images of object on the screen for two of its different positions, a distance x apart. The two real
images have magnifications m 1 and m 2 respectively (m 1 > m 2).
(A)

x
m1 m 2

(B) m 1 m 2 = 1

(C)

D2 x 2
4D

(D) all the above

www.physicsashok.in

12

OPTICS
26. A liquid of refractive index 1.33 is placed between two identical
plano-convex lenses, with refractive index 1.50. Two possible
arrangement P and Q are shown. The system is
(A) divergent in P, convergent in Q.
(B) convergent in P, divergent in Q.
(C) convergent in both
(D) divergent in both.

27. A lens of refractive index is put in a liquid of refractive index . If the focal length of the lens in air is

, its focal length in liquid will be


(A)


(B) 1

1
(C)

(D)

28. A convergent lens is placed inside a cell filled with a liquid. The lens has a focal length +20 cm when in
air and its material has a refractive index 1.50. If the liquid has a refractive index 1.60, the focal length of
the system
(A) 160 cm
(B) 24 cm
(D) 80 cm
(D) + 80 cm
29. A double convex lens, made of glass of refractive index 1.5, has focal length 6 cm. The radius of curvature
of one surface is double than that of other surface. The small radius of curvature has value
(A) 4.5 cm
(B) 6 cm
(C) 4 cm
(D) 9 cm
30. If the distance between a projector and screen is increased by 1%, then illumination on the screen
decreases by
(A) 1 %
(B) 2 %
(C) 3 %
(D) 4 %

31. A lens forms a sharp image of a real object on a screen. On inserting a parallel slide between the lens
and the screen with its thickness along the principal axis of the lens it is found necessary to shift the
screen parallel to itself d away from the lens for getting image sharply focused on it. If the refractive
index of the glass relative to air is , the thickness of slab is
(A)

(B) d

(C)

d
1

(D) 1

32. A thin convex lens in used to form a real image of a bright point object. The
apeture of the lens is small. A graph, shown is obtained by plotting a suitable
parameter Y against another suitable parameter x.
If

= the focal length of the lens


u = object distance
v = image distance
and Real Positive Convention is used then
(A) (uV) x; (u + V) y
(B) (u + V) x; (uV) y
1
u
1
(C) u x;
(D)
y
x;
y
u
v
v

Y
O

-1

www.physicsashok.in

13

OPTICS
33. Which of the following best represents object distance u vs image distance v graph for a convex lens.
y
y
y
y

(A)

(B)

(C)

(D)

34. Three thin prisms are combined as shown in figure. The refractive indices of the crown glass for red,
yellow and violet rays are r, y and v respectively and those for the flint glass are r, y and v
respectively. The ratio A/A for which there is no net angular dispersion.

2 y 1
(A)

y 1

y 1 y
(C) 1
y
y

y y
(B) 2
y y

2 y . y
(D)
y
y

35. A point object is placed at distance of 0.3 m from a convex lens of focal length
0.2 m cut into two equal halves, each of which is displaced by 0.0005 m, as shown
in figure. If C1 and C2 be their optical centres then,
(A) an image is formed at a distance of 0.6 m from C1 or C2 along principal axis.

C1
O

(B)

two images are formed, one at a distance of 0.6 m and other at a distance
of 1.2 m from C1 or C2 along principal axis.

(C)
(D)

an image is formed at a distance of 0.12 m from C1 or C2 along principal axis.


two images are formed at a distance of 0.6 m from C1 or C2 along principal axis

C2

at a separation of 0.003 m.

36.

A glass prism of refractive index 1.5 is immersed in water (refractive index 4/3). A light beam incident
normally on the face AB is totally reflected to reach on the face BC if
(1983)
A
B

(A) sin
37.

8
9

(B)

2
8
sin
3
9

(C) sin

2
3

A ray of light from a denser medium strike a rarer medium at an angle of incidence i (see Figure). The
reflected and refracted rays make an angle of 90 with each other. The angles of reflection and refraction
are r and r The critical angle is
r

r'

(A) sin

tan r

(B) sin

tan i

(C) sin

tan r

(D) tan

www.physicsashok.in

sin i
14

OPTICS
38.

Two coherent monochromatic light beams of intensities and 4 are superposed. The maximum and
minimum possible intensities in the resulting beam are
(A) 5 and
(B) 5 and 3
(C) 9 and
(D) 9 and 3

39.

An isosceles prism of angle 120 has a refractive index 1.44. The parallel monochromatic rays enter
the prism parallel to each other in air as shown. The rays emerge from the opposite faces

120

(A) are parallel to each other


(B) are diverging
(C) make an angle 2 [sin1 (0.72) 30] with each other
(D) make an angle 2 sin1 (0.72) with each other
40.

A diminished image of an object is to be obtained on a screen 1.0 m from it. This can be achieved by
appropriately placing
(A) a concave mirror of suitable focal length
(B) a convex mirror of suitable focal length
(C) a convex lens of focal length less than 0.25 m
(D) a concave lens of suitable focal length

41.

A concave lens of glass, refractive index 1.5 has both surfaces of same radius of curvature R. On
immersion in a medium of refractive index 1.75, it will behave as a
(A) convergent lens of focal length 3.5 R
(B) convergent lens of focal length 3.0 R
(C) divergent lens of focal length 3.5 R
(D) divergent lens of focal length 3.0 R

42.

A hollow double concave lens is made of very thin transparent material. It can be filled with air or either
of two liquids L1 and L2 having refractive indices

1 and 2 respectively 2 1 1 . the lens will

diverge a parallel beam of light if it is filled with


(A) air and placed in air
(B) air and immersed in L1
(C) L1 and immersed in L2
(D) L2 and immersed in L1
43.

A diverging beam of light from a point source Is having divergence angle , falls symmetrically on a
glass slab as shown. The angles of incidence of the two extreme rays are equal. If the thickness of the
glass slab is t and the refractive index n, then the divergence angle of the emergent beam is

(A) zero

44.

(B)

(C) sin

(D) 2sin

1

n

A ray of light passes through four transparent media with refractive indices

1 , 2 , 3 and 4 as

shown in the figure. the surfaces of all media are parallel. If the emergent ray CD is parallel to the
incident ray AB, we must have

(A)

1 2

(B)

2 3

(C)

3 4

(D)

4 1

www.physicsashok.in

15

OPTICS
45.

A given ray of light suffers minimum deviation in an equilateral prism p, Additional prism Q and R of
identical shape and of the same material as P are now added as shown in the figure. The ray will now
suffer

(A) greater deviation


(C) same deviation as before
46.

(B) no deviation
(D) total internal reflection

Which one of the following spherical lenses does not exhibit dispersion? The radii of curvature of the
surfaces of the lenses are as given in the diagrams.
(A) R1

47.

R2

(B) R

(C) R

(D) R

Two plane mirrors A and B are aligned parallel to each other, as shown in the figure A light ray is
incident at an angle 30 at a point just inside one end of A. The plane of incidence coincides with the
plane of the figure. The maximum number of times the ray undergoes reflections (including the first
one) before it emerges out is

(A) 28

(B) 30

(C) 32

(D) 34

48.

The size of the image of an object, which is at infinity, as formed by a convex lens of focal length 30 cm
is 2 cm. If a concave lens of focal length 2 0 cm is placed between the convex lens and the image at a
distance of 26 cm from the convex lens, calculate the new size of the image.
(A) 1.25 cm
(B) 2.5 cm
(C) 1.05 cm
(D) 2 cm

49.

A ray of light is incident at the glass-water interface at an angle i, it emerges finally parallel to the
surface of water, then the value of

(A) 4 3 sin i
50.

(B)

1 sin i

g would be

(C)

43

(D) 1

A beam of white light is incident on glass air interface from glass to air such that green light just suffers
total internal reflection. The colors of the light which will come out to air are
(A) Violet, Indigo, Blue
(B) All colors except green
(C) Yellow, Orange, Red
(D) White light

www.physicsashok.in

16

OPTICS
51.

An equilateral prism is placed on a horizontal surface. A ray pQ is incident onto it. For minimum deviation
R
S

Q
P

(A) PQ is horizontal
(C) RS is horizontal

(B) QR is horizontal
(D) Any one will be horizontal

52.

A source emits sound of frequency 600 Hz inside water. The frequency heard in air will be equal to
(velocity of sound in water = 1500 m/s, velocity of sound in air = 300 m/s)
(A) 3000 Hz
(B) 120 Hz
(C) 600 Hz
(D) 6000 Hz

53.

A point object is placed at the centre of a glass sphere of radius 6 cm and refractive index 1.5. The
distance of virtual image from the surface is
(A) 6 cm
(B) 4 cm
(C) 12 cm
(D) 9 cm

54.

A convex lens is in contact with concave lens. the magnitude of the ratio of their focal length is

2 3.

Their equivalent focal length is 30 cm. What are their individual focal lengths?
(A) 15, 10
(B) 10, 15
(C) 75, 50
(D) 75, 50
55.

A container is filled with water

1.33 upto a height

of 33.25 cm. A concave mirror is placed 15 cm above


the water level and the image of an object placed at
the bottom is formed 25 cm below the water level.
Focal length of the mirror is
(A) 15 cm
(B) 20 cm
(C) 18, 31 cm
(D) 10 cm

Multiple Choice Question with ONE or MORE THAN ONE correct answer:
56.

A convex lens of focal length 40 cm is in contact with a concave lens of focal length 25 cm. The power
of the combination is
(A) 1.5 dioptres
(B) 6.5 dioptres
(C) +6.5 diopres
(D) +6.67 dioptres

57.

A converging lens is used to form an image on a screen. When the upper half of the lens is covered by
an opaque screen
(A) half the image will disappear
(B) complete image will be formed
(C) intensity of the image will increase
(D) intensity of the image will decrease.

58.

A short linear object of length b lies along the axis of a concave mirror of focal length f at a distance u
from the pole of the mirror. The size of the image is approximately equal to
12

u f
(A) b

f
59.

12

f
(B) b

u f

u f
(C) b

f
(D) b

u f

A beam of light consisting of red, green and blue colours is incident on a right angled prism, figure. The
refractive indices of the material of the prism for the above red, green and blue wavelengths are 1.39,
1.44 and 1.47 respectively. The prism will

www.physicsashok.in

17

OPTICS

45

(A) separate part of the red colour from the green and blue colours
(B) separate part of the blue colour from the red and green colours
(C) separate all the three colours from one another.
(D) not separate even partially any colour from the other two colours.
60.

A thin prism P1 with angle 4 and made from glass of refractive index 1.54 is combined with another
thin prism P2 made from glass of refractive index 1.72 to produce dispersion without deviation. The
angle of the prism P2 is
(A) 5.33
(B) 4
(C) 3
(D) 2.6

61.

Two thin convex lenses of focal lengths f 1 and f 2 are separated by a horizontal distance d (where

d f1 d f 2 ) and their centres are displaced by a vertical separation as shown in Figure.

Taking the origin of coordinates O, at the center of the first lens the x and y coordinates of the focal
point of this lens system, for a parallel beam of rays coming from the left, are given by:
(A) x

f1 f 2
,y
f1 f 2

(B) x

f1 f 2 d

,y
f1 f 2 d
f1 f 2

(C) x

f1 f 2 d f1 d
f1 d
,y
f1 f 2 d
f1 f 2 d

(D) x

f1 f 2 d f1 d
,y0
f1 f 2 d

62.

Which of the following form(s) a virtual and erect image for all positions of the object?
(A) Convex lens
(B) Concave lens
(C) Convex mirror
(D) Concave mirror.

63.

A ray of light travelling in a transparent medium falls on a surface separating the medium from air at an
angle of incidence of 45. The ray just undergoes total internal reflection. If n is the refractive index of
the medium with respect to air, select the possible value(s) of n from the following:
(A) 1.3
(B) 1.4
(C) 1.5
(D) 1.6

64.

A concave mirror is placed on a horizontal table, with its axis directed vertically upwards. Let O be the
pole of the mirror and C its centre of curvature. A point object is placed at C. It has a real image, also
located at C. If the mirror is now filled with water, the image will be.
(A) real, and will remain at C
(B) real, and located at a point between C and
(C) virtual, and located at a point between C and O
(D) real, and located at a point between C and O

www.physicsashok.in

18

OPTICS

Fill in the blanks:


1.

A light wave of frequency 5 x 1014 Hz enters a medium of refractive index 1.5, In the medium the
velocity of the light wave is .................. and its wavelength is ................
(2 Marks)

2.

A convex lens A of focal length 20 cm and a concave lens B of focal length 5 cm are kept along the
same axis with a distance d between them. If a parallel beam of light falling on A leaves B as a parallel
beam, then d is equal to .......... cm.

3.

A monochromatic beam of light of wavelength 6000 in vacuum enters a medium of refractive index
1.5. In the medium its wavelength is ..., its frequency is .............
(1985)

4.

In youngs double-slit experiment, the two slits act as coherent sources of equal amplitude A and of
wavelength . In another experiment with the same set-up the two slits are sources of equal amplitude
A and wavelength , but are incoherent. The ratio of the intensity of light at the midpoint of the
screen in the first case to that in the second case is .................
(1986)

5.

A thin lens of refractive index 1.5 has 7a focal length of 15 cm in air. when the lens is placed in a
medium of refractive index

4
, its focal length will become ........... cm. (1987)
3

6.

A point source emits sound equally in all directions in a non-absorbing medium. Two points P and Q
are at a distance of 9 meters and 25 meters respectively from the source. The ratio of amplitudes of
the waves at P and Q is ....................
(1989)

7.

A slab of a material of refractive index 2 shown in Figure, has a curved surface APB of radius of
curvature 10 cm and a plane surface CD. On the left of APB is air and on the right of CD is water with
refractive indices as given in the figure.

n1=1.0

C
n2=2.0
C

n3 =

4
3

15 cm
B

D
20 cm

An object O is placed at a distance of 15 cm from the pole P as shown. The distance of the final image
of O from P, as viewed from the left is ...............
(1991)

8.

A thin rod of length

f
is placed along the optic axis of a concave mirror of focal length f such that itss
3

image which is real elongated, just touches the rod. The magnification is .............
9.

A ray of light undergoes deviation of 30 when incident on an equilateral prism of refractive index
The angle made by the ray inside the prism with the base of the prism is .............

10.

(1991)

2.

(1992)

A light of wavelength 6000 in air, enters a medium with refractive index 1.5. Inside the medium its
frequency is ........ Hz and its wavelength is ............... .
(1997)

www.physicsashok.in

19

OPTICS
11.

Two thin lenses, when in contact, produce a combination of power +10 diopters. When they are 0.25 m
apart, the power reduces to +6 diopters. The focal length of the lenses are ...... m and .... m.
(1997)

12.

A ray of light is incident normally on one of the faces of a prism of apex angle 30 and refractive index

2 . The angle of deviation of the ray is .......... degrees.

(1997)

True / False :
13.

The intensity of light at a distance r from the axis of a long cylindrical source is inversely proportional
to r.
(1981)

14.

A convex lens of focal length 1 meter and a concave lens of focal length 0.25 meter are kept 0.75
meter apart. A parallel beam of light first passes through the convex lens, then through the concave
lens and moves to a focus 0.5 m away from the concave lens.
(1983)

15.

A beam of white light passing through a hollow prism give no spectrum.

16.

A parallel beam of white light fall on a combination of a concave and a convex lens, both of the same
material. Their focal lengths are 15 cm and 30 cm respectively for the mean wavelength in white light.
On the other side of the lens system, one sees coloured patterns with violet colour at the outer edge.
(1988)

17.

Match List I and List II and select the correct answer using the codes given below the lists:
The arrangement shows different lenses made of substance of refractive index 1.5 and kept in air. R1
= 30 cm, R2 = 60 cm. Match the focal lengths

(1983)

Table Match

Table I

I.

II.

III.

IV.

R1

R1

R1

R1

Table II
R2

R2

R2

R2

(A) I-A, II-B, III-D, IV-C


(C) I-D, II-C, III-A, IV-B

A.

120 cm

B.

+40 cm

C.

40 cm

D.

+120 cm

(B) I-C, II-A, III-B, IV-D


(D) I-B, II-D, III-C, IV-A

www.physicsashok.in

20

OPTICS
18.

Table I

Table II

I.

An object is placed at focus before


A.
Magnification is
a convex mirror
II.
An object is placed at the centre of
B.
Magnification is +0.5
curvature before a concave mirror
III.
An object is placed at focus before
C.
Magnification is +1
a concave mirror.
IV.
An object is placed at centre of curvature
D.
Magnification is 1
before a convex mirror.
(A) I-B, II-D, III-A, IV-E
(B) I-A, II-D, III-C, IV-B
(C) I-C, II-B, III-A, IV-E
(D) I-B, II-E, III-D, IV-C

19.

Match the followings:

Table I

Table II

A.

Magnification m = +1

(i)

Convex mirror

B.

Magnification m

2
3

(ii)

Plane mirror

C.

Magnification m

3
2

(iii)

Concave mirror

(A) A (ii) B (iii) C (i)


(C) A (ii) B (i) C (iii)

20.

(B) A (i) B (ii) C (iii)


(D) A (iii) B (ii) C (i)

For a concave mirror of focal length 20 cm, match the followings:

Table I
Objective distance
A.
10 cm
B.
30 cm
C.
40 cm
D.
50 cm
(A) A II, B I, C III, D IV
(C) A I, B IV, C III, D II

Table II
Nature of image
(i)
Magnified, inverted and real
(ii)
Equal size, inverted and real
(iii) Smaller, inverted and real
(iv) Magnified, erect and virtual
(B) A IV, B I, C II, D III
(D) A IV, B I, C III, D II.

PASSAGE TYPE QUESTIONS


THE NEXT QUESTIONS REFER TO THE FOLLOWING PASSAGE
The ciliary muscles of eye control the curvature of the lens in the eye and hence can alter the effective
focal length of the system. When the muscles are fully relaxed, the focal length is maximum. When the
muscles are strained the curvature of lens increases (that means radius of curvature decreases) and
focal length decreases. For a clear vision, the image must be on retina. The image distance is therefore
fixed for clear vision and it equals the distance of retina from eye-lens. It is about 2.5 cm for a grownup person.

www.physicsashok.in

21

OPTICS
A person can theoretically have clear vision of objects situated at any large distance from the eye. The
smallest distance at which a person can ciliary muscles are most strained in this position. For an
average grown-up person, minimum distance of object should be around 25 cm.
A person suffering for eye defects uses spectacles (eye glass). The function of lens of spectacles is
to form the image of the objects within the range in which person can see clearly. The image of the
spectacle lens becomes object for eye-lens and whose image is formed on retina. The number of
spectacle lens used for the remedy of eye defect is decided by the power of the lens required and the
number of spectacle-lens equal to the numerical value of the power of lens with sign. For example,
power of lens required is +3 D (converging lens of focal length

100
cm), then number of lens will be
3

+3.
For all the calculations required you can use the lens formula and lens makers formula. Assume that
the eye lens is equiconvex lens. Neglect the distance between eye lens and the spectacle lens.
1.

Minimum focal length of eye-lens of a normal person is


(A) 25 cm

2.

(B) 2.5 cm

25
cm
9

(C)

(D)

25
cm
11

(D)

25
cm
11

Maximum focal-length of eye lens of normal person is


(A) 25 cm

(B) 2.5 cm

25
cm
9

(C)

3.

A near-sighted man can clearly see object only upto a distance of 100 cm and not beyond this. The
number of the spectacles lens necessary for the remedy of this defect will be
(A) + 1
(B) 1
(C) + 3
(D) 3

4.

A far-sighted man cannot see object only upto a distance of 100 cm from his eyes. The number of the
spectacles lens that will make his range of clear vision equal to an average grown up person is
(A) + 1
(B) 1
(C) + 3
(D) 3

5.

A person who can see objects clearly from distance 10 cm to , then we can say that the person is
(A) normal sighted person
(B) near-sighted person
(C) far-sighted person
(D) a person with exceptional eyes having no eye defect.
THE NEXT QUESTIONS REFER TO THE FOLLOWING PASSAGE
Spherical aberration in spherical mirrors is a defect which is due to dependence of focal length ' f '
on angle of incidence

' ' as shown in figure. is given by


f R

R
sec
2

Pole (P)

Principal axis

F
f

where R is radius of curvature of mirror and is the angle of incidence. The rays which are closed to
principal axis are called paraxial rays and the rays far away from principal axis are called marginal
rays. As a result of above dependence different rays are brought to focus at different points and the
image of a point object is not a point.

www.physicsashok.in

22

OPTICS
6.

If f p and f m represent the focal length of paraxial and marginal rays respectively, then correct relationship
is:
(A) f p f m

7.

(B) f p f m

(C) f p f m

(D) None

If angle of incidence is 60, then focal length of this rays is:


(A) R

(B)

R
2

(C) 2R

(D) 0

8.

The total deviation suffered by the ray falling on mirror at an angle of incidence equal to 60 is:
(A) 180
(B) 90
(C) Cant be determined
(D) None

9.

For paraxial rays, focal length approximately is:


(A) R

10.

(B)

R
2

(C) 2R

(D) None

Which of the following statements are correct regarding spherical aberration:


(A) It can be completely eliminated
(B) it cant be completely eliminated but is cant be minimised by allowing either paraxial or marginal
rays to hit the mirror
(C) It is reduced by taking large aperture mirrors
(D) None
THE NEXT QUESTIONS REFER TO THE FOLLOWING PASSAGE
Rainbow is formed during rainy season due to refraction and total internal reflection of rays falling on
suspended water droplets. When rays of the sun fall on rain drops, the rain drops disperse the light and
deviate the different colours by refraction and total internal reflection to the eye of the observer. A
person observing the drops will see different colours of the spectrum at different angles. The rainbow
which results from single total internal reflection is called primary rainbow and secondary rainbow is
formed due to two total internal reflections suffered by rays falling on water drops.

Rays from
sun

A
V

C
R

Re
d

D
l et
vio

11.

12.

Secondary
rainbow

R
Re
d

v io
le t

Primary
rainbow

Figure shows formation of rainbow due to four drops A, B, C and D. The light surffers only one total
linternal reflection in drops C and D forming primary rainbow. Secondary rainbow is formed by drops A
and B where light suffers two total linternal reflections.
Rainbow is an arc of:
(A) Circle
(B) Ellipse
(C) Parabola
(D) Cant be determined
The visibility of the rainbow is due to:
(A) All rays
(B) Rays undergoing maximum deviation
(C) Rays undergoing minimum deviation
(D) None

www.physicsashok.in

23

OPTICS
13.

14.

15.

In primary rainbow, the colour of outer edge is:


(A) Blue
(B) Violet
(C) Red

(D) None

In secondary rainbow, the colour of inner edge is:


(A) Red
(B) Violet
(C) Indigo

(D) None

The necessary condition for the observer to see rainbow is:


(A) Sun, observers eye and the centre of the rainbow arc lie on the same line
(B) Sun, observers eye and the centre of the rainbow arc lie on the different line
(C) From any position provided sun is at the back of the observer
(D) None
THE NEXT QUESTIONS REFER TO THE FOLLOWING PASSAGE
The laws governing the behavior of the rays namely rectilinear propagation, laws of reflection and
refraction can be summarised in one fundamental also known as Fermats principle. According to this
principle a ray of light travels from one point to another such that the time taken is at a stationary value
(maximum or minimum). If c is the velocity of light in a vacuum, the velocity in a medium of refractive
index n is

c
nl
, hence time taken to travel a distance l is
. If the light passes through a number of
n
c

1
1
nl and n dl . If refractive index varies continuously. Now,, nl
c
c

media, the total time taken is

is the total optical path, so that Fermats principle states that the path of a ray is such that the optical
path in at a stationary value. This principle is obviously in agreement with the fact that the ray are
straight lines in a homogenous isotropic medium. It is found that it also agrees with the classical laws of
reflection and refraction.
16.

If refractive index of a slab varies as

1 x 2 where x is measured from one end, then optical path

length of a slab of thickness 1 m is:


(A)
17.

4
m
3

(B)

3
m
4

(C) 1 m

(D) None

The optical path length followed by ray from point A to B given that laws of reflection are obeyed as
shown in figure is:
A

P
(A) Maximum
18.

(B) Minimum

(C) Constant

(D) None

The optical path length followed by ray from point A to B given that laws of reflection are obeyed as
shown in figure is
A

(A) Maximum

(B) Minimum

(C) Constant

(D) None

www.physicsashok.in

24

OPTICS
19.

The optical path length followed by ray from point A to B given that laws of refraction are obeyed as
shown in figure is
A

B
(A) Maximum
20.

(B) Minimum

(C) Constant

(D) None

The optical path length followed by ray from point A to B given that laws of refraction are obeyed as
shown in figure is

A
B
A and B are focii
of ellipse
(A) Maximum

(B) Minimum

(C) Constant

(D) None

THE NEXT QUESTIONS REFER TO THE FOLLOWING PASSAGE


One hard and stormy night you find yourself lost in the forest when you come upon a small hut.
Entering it you see a crooked old woman in the corner hunched over a crystal ball. You are about to
make a hasty exit when you hear the howl of wolves outside. Taking another look at the gypsy you
decide to take your chances with the wolves, but the door is jammed shut. Resigned to bad situation
your approach her slowly, wondering just what is the focal length of that nifty crystal ball.
21.

If the crystal ball is 20 cm in diameter with R.I. = 1.5, the gypsy lady is 1.2 m from the central of ball,
where is the image of the gypsy in focus as you walk towards her?
(A) 6.9 cm from the crystal ball
(B) 7.9 cm from the crystal ball
(C) 8.9 cm from the crystal ball
(D) None

22.

The image of old lady is:


(A) real, inverted an enlarged
(C) erect, virtual and magnified

23.

(B) erect, virtual and small


(D) real, inverted and diminished

The old lady moves the crystal ball closer to her wrinkled old face. At some point you can no longer get
an image of her. At what object distance will there be no change of the gypsy formed?
(A) 10 cm
(B) 5 cm
(C) 15 cm
(D) None

THE NEXT QUESTIONS REFER TO THE FOLLOWING PASSAGE


The table below contains some physical properties of common optical materials. The refractive index
of a material is a measure of the amount by which light is bent upon entering the material. The
transmittance range is the range of wavelengths over which the material is transparent.

www.physicsashok.in

25

OPTICS

Material

Lithium
fluoride
Calcium
fluoride
Sodium
chloride
Quartz
Potassium
bromide
Flint glass*
Cesium iodide

Physical Properties of Optical Materials


Refractive
Transmittance
Useful range
index for light
for prisms
range (m)
of 0.589 m
(m)
1.39
0.12-6
2.7-5.5

Chemical
resistance
Poor

1.43

0.12-12

5-9.4

Good

1.54

0.3-17

816

Poor

1.54
1.56

0.20-3.3
0.3-29

0.20-2.7
1528

Excellent
Poor

1.66
0.352.2
0.35-2
1.79
0.37.0
15-55
*Flint glass is lead oxide doped quartz.

Excellent
Poor

24.

According to the table, which material(s) will transmit light at 25 m


(A) Potassium bromide only
(B) Potassium bromide and cesium iodide
(C) Lithium fluoride and cesium iodide
(D) Lithium fluoride and flint glass

25.

A scientist hypothesizes that any material with poor chemical resistance would have a transmittance
range wider than 10 m . The properties of which of the following materials contradicts this hypothesis
(A) Lithium fluoride
(B) Flint glass
(C) Cesium iodide
(D) Quartz

26.

When light travels from one medium to another, total internal reflection can occur if the first medium
has a higher refractive index than the second. Total internal reflection could occur if light were travelling
from
(A) Lithium fluoride of flint glass
(B) potassium bromide to cesium iodide
(C) quartz to potassium bromide
(D) flint glass to calcium fluoride

27.

Based on the information in the table, how is the transmittance range related to the useful prism
range
(A) The transmittance range is always narrower than the useful prism range
(B) The transmittance range is narrower than or equal tot he useful prism range
(C) The tranmittance range increases as the useful prism range decreases
(D) The tranmittance range is wider than and includes within it the useful prism range

28.

The addition of lead oxide to pure quartz has the effect of


(A) decreasing the transmittance range and the refractive index
(B) decreasing the transmittance range and increasing the refractive index
(C) increasing the transmittance range and the useful prism range
(D) increasing the transmittance range and decreasing the useful prism range.
THE NEXT QUESTIONS REFER TO THE FOLLOWING PASSAGE
A periscope viewing system is to be used to observe the behavior of primates in a large environmentally
controlled room on the upper floor of a large research facility. The periscope, like those used on

www.physicsashok.in

26

OPTICS
submarines, is essentially a large, folded-path, low power telescope (using prisms to fold the light path).
A sketch of the preliminary design appears below. Like all Newtonian telescopes, it uses a relatively long
focal length objective lens to form a real image in front of the eyepiece lens (of shorter focal length). The
observer looks through the eyepiece lens to see the final image, in the same manner that one would use
a magnifying glass.

The distance between the lenses is approximately equal to the sum of their focal lengths. The eyepiece,
in this design, can be moved forward or back in order to focus on the primates as they move closer to
or further away from the objective lens.
29.

The total tube length of the three sections is to be 4 m. The objective lens available has a focal length
of 3 m. What should the focal length of the eyepiece lens be?
(A) 0.75 m
(B) 1 m
(C) 1.33 m
(D) 7 m

30.

A visitor seeing the sketch points out an important flaw that will require a design change. what is the
flaw?
(A) The focal length of the eyepiece lens is too short.
(B) The images of the primates will be inverted
(C) The objective lens should be a diverging lens.
(D) The prisms cannot be used in this way.

31.

A visitor seeing the sketch points out an important flaw that will require a design change. What is the
flaw?
(A) The focal length of the eyepiece lens is too short.
(B) The images of the primates will be inverted
(C) The objective lens should be a diverging lens.
(D) The prisms cannot be used in this way.

32.

What will be the approximate magnification of this periscope?


(A) 0.67x
(B) 1x
(C) 3x
(D) 300x

33.

The prisms (454590 prisms) turn the light path through 90 by total internal reflection from the
inside hypotenuse faces of the prisms when the incident angle is 45 as in the sketch.
Can one use crown glass with an index of refraction of 1.52 for the prism?
(A) yes, because the critical angle for crown glass is 47
(B) yes, because the critical angle for crown glass is 41.
(C) No, because the critical angle for crown glass is exactly 47
(D) No, because the critical angle for crown glass is exactly 41.

34.

Describe the properties of the image that one sees with this preliminary design
(A) real, inverted, magnified
(B) real, upright, magnified
(C) virtual, upright, same size as object.
(D) virtual, inverted, magnified

35.

The telescope is focused on a primate rather far away on the farside of the large habitat. As the
primate moves rather closer to the telescope, what must the observer do to see the primate clearly?
(A) No change, the image remains clear.
(B) Move the eyepiece away from the objective.
(C) Move the eyepiece closer to the objective.
(D) Use an inverting eyepiece because the image flips.

www.physicsashok.in

27

OPTICS
THE NEXT QUESTIONS REFER TO THE FOLLOWING PASSAGE
In the normal human eye, light from an object is refracted by the cornea-lens system at the front of the
eye and produces a real image on the retina at the rear of the eye. For a given eye, its lens-to-retina
distance is fixed at about 2.5 cm. Most of the focusing of an image is done by the cornea, which has a
fixed curvature that is convex with respect to incoming light. The importance of the lens is that its
radius of curvature ccan be changed, allowing the lens to fine-tune the focus.
The lens is surrounded by the ciliary muscle. Contraction of the muscle decreases tension on the
lens. This allows the natural elasticity of the lens to produce an increase in the radius of curvature.
when the muscle relaxes, the lens flattens out, decreasing tis radius of curvature. Unfortunately, the
lens losses elasticity with age and the ability to alter curvature decreases.
The range over which clear vision is possible is bounded by the far point and the near point. In normal
vision the far point is infinity and the near point depends on the radius of curvature of the lens. For
normal eyes the average near point for reading is 25 cm.

AGE, years
10
20
30
40
50
60

NEAR POINT, cm
7
10
14
22
40
200

In the myopic (nearsighted) eye, the lens-to-retina length is too long and/or the radius of curvature of
the cornea is too great. This causes rays from an object at infinity to focus at a point in front of the
retina. The far point is closer than normal .A corrective lens will put a virtual image of a distant object
at the position of the actual far point of the eye.
In the hyperopic (farsighted) eye, the lens-to-retina length is too short and/or the radius of curvature
of the cornea is not great enough. This causes rays from an object at infinity to focus at a point behind
the retina. The near point is farther away than normal. A corrective lens will put a virtual image of the
close object at the position of the actual near point.
The relation among the object (o) and image (i) distances from the eye and the focal length (f) of the
lens is given by the lens-distance rule :

1 o 1 i 1 f .

When using this equation, all distances are given in centimeters.


The power of corrective lenses is usually given in units called diopters. Power, in diopters, is the
reciprocal of the focal length in meters : Pdiopter 1 f meter .
By convection
I. Converging lenses have positive focal lengths, and diverging lenses have negative focal lengths.
II. Real images have positive distances from the lens, and virtual images have negative distances from
the lens.
36.

The lens system of the myopic eye is best described as


(A) producing too much convergence.
(B) producing too little convergence.
(C) producing too much divergence.
(D) producing too little divergence.

37.

An optometrist examined Johns eyes. The farthest object he can clearly focus on with his right eye is
50 cm away. What is the power of the contact lens required to correct the vision in his right eye
(A) 0.50 diopters
(B) 2.0 diopters
(C) +2.0 diopters
(D) +5.0 diopters

www.physicsashok.in

28

OPTICS
38.

In a mildly hyperopic eye, the focal length of the eyes natural lens can be corrected by
(A) contracting the muscle and increasing the radius of curvature.
(B) contracting the ciliary muscle and decreasing the radius of curvature
(C) relaxing the ciliary muscle and increasing the radius of curvature.
(D) relaxing the ciliary muscles and decreasing the radius of curvature.

39.

Jane must wear a contact lens with a power of +3.00 diopters in one eye to be able to clearly focus on
an object 2.5 cm in front of the eye. Based on the vision in this eye, which of the following is the most
likely age range for Jane
(A) Less than 40 years old
(B) From 40 to 49 years old
(C) From 50 to 59 years old
(D) 60 years or older

40.

George wears eyeglasses that sit 2.0 cm in front of his eyes. His incorrect far point is 50 cm. What is
the focal lengths of his eyeglasses
(A) 50 cm
(B) +50 cm
(C) 48 cm
(D) +48 cm

41.

In a surgical procedure called radial keratotomy, (RK), a laser is used to flatten the cornea by placing
as series of hairline cuts around the perimeter of the cornea. Which statement is most accurate
(A) RK corrects myopia by decreasing the focal length of the eye.
(B) RK corrects myopia by increasing the focal length of the eye.
(C) RK corrects hyperopia by decreasing the focal length of the eye.
(D) RK corrects hyperopia by increasing the focal length of the eye.
THE NEXT QUESTIONS REFER TO THE FOLLOWING PASSAGE
Student are given a variety of lenses and optics equipment, such as lens holders, lighted object
sources. Optical benches, meter sticks and tapes, image screens, and several examples of commercial
optical equipment, such as microscopes and telescopes. They are to work in an open-ended optics lab
in order to learn the general principles of lenses and the optical devices that can be constructed using
lenses.

42.

A student is given a short focal length converging lens and long focal length converging lens. One lens
is placed in a holder. A lighted object is placed 18 cm in front of the lens and it is found that a clear
image can be focused on a screen placed 36 cm behind the lens. what is the focal length of this lens?
(A) 8 cm
(B) 12 cm
(C) 27 cm
(D) 46 cm

43.

What magnification is produced by the above lens when the object is 18 cm in front of the lens and the
image is 36 cm behind the lens?
(A) 2x
(B) 3x
(C) 4x
(D) 6x

44.

A lighted object is placed 6 cm in front of the second lens, which has a focal length of +24 cm. Where
is the image and which kind of image is it?
(A) 8 cm in front of the lens: a virtual image.
(B) 8 cm behind the lens: a real image
(C) 16 cm in front of the lens: a real image
(D) 16 cm behind the lens; a virtual image.

45.

The 24 cm focal length lens is used as the objective of a simple refracting telescope and a third
converging lens of focal length +8 cm is used as the eyepiece. What is the magnification of this simple
refractor?
(A) 0.6x
(B) 3x
(C) 4x
(D) 6x

46.

A commercial microscope is examined by the student. The objective is marked 20x and the eyepiece
is marked 10x. what power objective should replace the above objective so that the microscopes
magnification will be 400x
(A) 5x
(B) 10x
(C) 40x
(D) 100x

www.physicsashok.in

29

OPTICS
47.

A lighted object is placed 24 cm in front of a +12 cm focal lengths lens. The image formed by this lens
is the object for a second lens of +24 cm focal length. The second lens is placed 72 cm behind the first
lens. where is the final image with respect to the second lens?
(A) 24 cm in front of # 2
(B) 24 cm behind # 2
(C) 36 cm in front of # 2
(D) 48 cm behind # 2

48.

A lens of focal length +24 cm is used to view an object placed 12 cm in front of the lens. The object is
5 cm tall. How tall is the image?
(A) 2.5 cm
(B) 3.3 cm
(C) 7.5 cm
(D) 10 cm

49.

A diverging lens of focal length 24 cm is now used with the object 12 cm in front of the lens. How tall
is the image if the object is 5 cm tall?
(A) 2.5 cm
(B) 3.3 cm
(C) 8 cm
(D) 10 cm

50.

A near sighted student cannot see objects clearly unless they are as close as 80 cm (his far-point).
The image that he sees through his new contact lens is a virtual image because he looks through the
lens to see the image. what focal length lenses does he need in order to see very distance objects,
such as the starts?
(A) 20 cm
(B) 30 cm
(C) 4 cm
(D) 25 cm
THE NEXT QUESTIONS REFER TO THE FOLLOWING PASSAGE
The phenomenon of refraction has long intrigued scientists and was actually used to corroborate
one of the major mysteries of early science: the determination of the speed of light.
The refractive index of a transparent material irrelated to a number of the physical properties of light.
In terms of velocity, the refractive index represents the ratio of the velocity of light in a vacuum to its
velocity in the material. From this ratio, it can be seen that light is retarded when it passes through
most types of matter. It is worth noting that prisms break up white light into the seven colors of the
rainbow because each color has a slightly different velocity in the medium.
Snells law allows one to follow the behavior of light in terms of its path when moving from a material
of one refractive index to another with the same, or different refractive index. It is given by:

n1 sin 1 n2 sin 2 , where I refers to the first medium through which the ray passes, 2 refers to the
second medium, and the angles refer to the angle of incidence in the first medium
of refraction in the second

1 and the angle

2 .

A ship went out on a search for a sunken treasure chest. In order to locate the chest, they shone a
beam of light down into the water using a high intensity white light source as shown in Figure. The
refractive index for sea water is 1.33 while that for air is 1.00.
51.

From the information in the passage, how would you expect the speed of light in air to compare with
the speed of light in a vacuum (which is given by c)?
(A) It would be the same (=c)
(B) It would be greater than c.
(C) It would be less than c.
(D) This cannot be determined from the information given.

www.physicsashok.in

30

OPTICS
52.

Using the information in the passage, what must the approximate value of
chest as shown in Figure?
(A) 15.2
(B) 30.4

53.

(C) 45.6

2 be such that it hit the

(D) 63.4

How does the refractive index in water light compare with that of red light given that violet light travels
more slowly in water than red light?
(A) nviolet nred

(B) nviolet nred

(C) nviolet nred

(D) This depends on the relative speeds of the different colors in a vacuum.

54.

Total internal reflection first occurs when a beam of light travels from one medium to another medium
which has a smaller refractive index at such an angle of incidence that the angle of refraction is 90.
This angle of incidence is called the critical angle. What is the value of the sine of this angle when the
ray moves from water towards air?
(A) 2
(B) 0.75
(C) 0.50
(D) 0

55.

What would happen to the critical angle, in the previous question, if the beam of light was travelling
from water to a substance with a greater refractive index than air, but a lower refractive index than
water?
(A) It would increase
(B) It would decrease
(C) It would remain the same
(D) Total internal reflection would not be possible.

56.

Which of the following would you expect to remain constant when light travels from one medium to
another and the media differ in their refractive indices?
(A) Velocity
(B) Frequency
(C) Wavelength
(D) Intensity.
THE NEXT QUESTIONS REFER TO THE FOLLOWING PASSAGE
The invention of the compound microscope by Jansen in the late 1500s truly revolutionized the world
of science, particularly the field of cellular and molecular biology. The discovery of the cell as the
fundamental unit of living organisms and the insight into the bacterial world are two of the contributions
of this instrument to science.
It is unseemly that such a relatively simplistic apparantus took generations to be developed. Its main
component are two convex lenses: one acts as the main magnifying lens and is referred to as the
objective, and another lens called the eyepiece. The two lenses act independently of each other when
bending light rays. The actual lens set-up depicted in Figure.

Light from the object (O) first passes thought he objective and an enlarged, inverted first image is
formed. The eyepiece then magnifies this image. Usually the magnification of the eyepiece is fixed
(either x 10 or x 10) and three rotating objective lenses are used : x 10, x 40 and x 60. The most recent
development in microscope technology is the electron microscope which uses a beam of electrons
instead of light. Photographic film must be used otherwise no image would be formed on the retina.
This microscope has a resolution about a hundred times that of the light microscope.
57.

Based on the passage, what type of image would have to be produced by the objective magnification?
(A) Either virtual or real
(B) Virtual
(C) Real
(D) It depends on the focal length of the lens.

www.physicsashok.in

31

OPTICS
58.

Where would the first image have to be produced by the objective relative to the eyepiece such that a
second, enlarged image would be generated on the same side of the eyepiece as the first image (first
image distance = d1)?
(A) di Fe

(B) di Fe

(C) 2 Fe d i Fe

(D) di 2 Fe

59.

Two compound microscopes A and B were compared. Both had objectives and eyepieces with the
same magnification but A gave an overall magnification that was greater than that of B. Which of the
following is a plausible explanation?
(A) The distance between objective and eyepiece in A is greater than the corresponding distance in B.
(B) The distance between objective and eyepiece in A is less than the corresponding distance is B.
(C) The eyepiece and objective positions were reversed in A.
(D) The eyepiece and objective positions were reversed in B.

60.

A student attempted to make a compound microscope. However, when she tried to view an object
through the apparatus, no image was seen. Which of the following could explain the mishap?
I. The object distance = focal length of objective.
II. The object distance for eyepiece lens as her eyepiece.
III. The student used a diverging lens as her eyepiece.
IV. The student used a converging lens as her objective
(A) I, II, III and IV
(B) I, II, III
(C) I, II, IV
(D) II, III, IV

61.

The magnification of the eyepiece of a compound microscope is x15. The image height is 25 mm and
the magnification of the objective is x40. What is the object height?
(A) 1.67 mm
(B) 0.60 mm
(C) 0.38 mm
(D) 0.04 mm

62.

What is the refractive power of an objective lens with a focal length of 0.50 cm?
(A) 0.2 diopters
(B) 2.0 diopters
(C) 20 diopters
(D) 200

THE NEXT QUESTIONS REFER TO THE FOLLOWING PASSAGE


Magnification by a lens of an object at distance 10 cm from it is 2. Now a second lens is placed
exactly at the same position where first was kept, without changing the distance object and lens. The
magnification by this second lens is 3.
63.

Now both the lenses are kept in contact at the same place. What will be the new magnification.
(A)

64.

13
5

(B)

12
7

(C)

6
11

(D)

5
7

What is the focal length of the combination when both lenses are in contact.
(A)

60
cm
17

(B)

5
cm
17

(C)

12
cm
7

(D)

13
cm
9

THE NEXT QUESTIONS REFER TO THE FOLLOWING PASSAGE


In the case of convex lens, when object is moved from f to 2f, its image is real, inverted and magnified.
It moves from f to infinity on other side.
65.

Focal length of a convex lens is 10 cm. When the object is moved from 15 cm to 25 cm, the magnitude
of linear magnifications.
(A) will increase
(B) will decrease
(C) will first increase then decrease
(D) will first decrease than increase.

www.physicsashok.in

32

OPTICS
66.

Image of object AB shown in figure will be like:


B
2F

A'
(A)

A'

(B)

2F

2F

B'
A'

(C)

A'

(D)

2F

B'

2F

B'

B'

THE NEXT QUESTIONS REFER TO THE FOLLOWING PASSAGE


Figure shows a simplified model of the eye that is based on the assumption that all of the refraction
of entering light occurs at the cornea. The cornea is a converging lens located at the outer surface of
the eye with fixed focal length approximately equal to 2 cm. Parallel light rays coming from a very
distant object are refracted by the cornea to produce a focused image on the retina. The retina then
transmits electrical impulse along the optic nerve to the brain.
cornea
retina

Two common defects of vision are myopia and hyperopia. Myopia, sometimes referred to as
nearsightedness, occurs when the cornea focuses the image of a distance object in front of the retina.
Hyperopia, sometimes referred to as farsightedness, occurs when the cornea focuses the image of a
nearby object behind the retina. Both of these problems can be corrected by introducing another lens
in front of the eye so that the two lens system produces a focused image on the retina. If an object is
so far away from the lens system that its distance may be taken as infinite, then the following relationship
holds:

1
1
1

, where f is the focal length of the cornea, f , is the focal length of the correcting
c
l
f c fl x v

lens, x is the distance from the correcting lens to the cornea, and v is the image distance measured
from the cornea. (Note : The index of refraction is 1.0 for air and 1.5 for glass).
67.

How far away should the retina be from the cornea for normal vision?
(A) 0.5 cm
(B) 1.0 cm
(C) 2.0 cm

(D) 4.0 cm

68.

For a distant object, the image produced by the cornea is:


(A) real and inverted
(B) real and upright
(C) virtual and inverted
(D) virtual and upright.

69.

What kind of lens would be suitable to correct myopia and hyperopia respectively? (Note : Assume that
the correcting lens is at the focal point of the cornea so that x f c .)
(A) Converging, converging
(C) Diverging, diverging

70.

(B) Converging, diverging


(D) Diverging, converging

The focal length of a womans cornea is 1.8 cm, and she wears a correcting lens with a focal length of
16.5 cm at a distance x = 1.5 cm from her cornea. What is the image distance v measured from the
cornea for a distant object?
(A) 1.0 cm
(B) 1.5 cm
(C) 2.0 cm
(D) 2.5 cm

www.physicsashok.in

33

OPTICS
71.

In the case of contact lens, the cornea and the correcting lens are actually touching and act together
as a single lens. If the focal length of both the cornea and the contact lens are doubled, then the image
distance v for a distant object would:
(A) be 1/4 the old value
(B) be 1/2 the old value.
(C) be the same as the old value
(D) be twice the old value.

Level # 2
A

1.

In what direction should a beam of light be sent from point A (Figure) contained in
a mirror box for it to fall onto point B after being reflected once from all four walls?
Point A and B are in one plane perpendicular to the walls of the box (i.e., in the
plane of the drawing).

2.

A concave mirror has the form of a hemisphere with a radius R = 55 cm. A thin layer of an unknown
transparent liquid is poured into this mirror, and it was found that the given optical system produces, with
the source in a certain position two real image, one of which (formed by direct reflection) coincide with
source and the other is at a distance of = 30 cm from it. Find the refractive index of the liquid.

3.

A point source of light S is placed on the major optical axis of concave mirror at a distance of 60 cm. At
what distance from the concave mirror should a flat mirror be placed for the rays to converge again at
the point S having been reflected from the concave mirror and then from the flat one? Will the position of
the point where the rays meet change if they are first reflected from the flat mirror? The radius of the
concave mirror is 80 cm.

4.

A pile 4 m high driven into the bottom of a lake is 1 m above the water.
Determine the length of the shadow of the pile on the bottom of the lake if the sun rays make an angle of
45 with the water surface. The refractive index of water is 4/3.

5.

In figure, a fish water watches a fish through a 3.0 cm thick glass wall of a fish tank. The watcher is in
level with the fish; the index of refraction of the glass is 8/5 and that of the water is 4/3.

8.0 cm

3.0 cm

6.8 cm

Observer
Water

Wall

(a)
(B)

6.

To the fish, how far away does the watcher appear to be?
To the watcher, how far away does the fish appear to be?

A hollow sphere of glass of refractive index has a small mark on its interior surface which is observed
from a point outside the sphere on the side opposite the center. The inner cavity is concentric with
external surface and the thickness of the glass is every where equal where equal to the radius of the

1 R
inner surface. Prove that the mark will appear nearer than it really is, by a distance 3 1 , where R is
the radius of the inner surface.

www.physicsashok.in

34

OPTICS
7.

8.

A long rectangular slab of transparent medium of thickness d


is placed on a table with length parallel to the x-axis and width
parallel to the y-axis.
A ray of light is travelling along y-axis at origin. the refractive
0
index of the medium varies as
, where 0 and
1 x r
r (> 1) are constants. The refractive index of air is 1.
(a) Determine the x-coordinate of the point A, where the ray
intersects the upper surface of the slab-air boundary.
(B) Write down the refractive index of the medium at A.
(C) Indicate the subsequent path of the ray in air.

A
d
medium
X

A man of height 2.0 m is standing on level road where because of temperature variation the refractive
index of air is varying as 1 ay , where y is height from road. If a = 2.0 x 106 m 1. Then find distant
point that he can see on the road.

9.

A portion of straight glass rod of diameter 4 cm and refractive index 1.5 is


bent into an arc of circle of radius R. A parallel beam of light is incident on
it as shown in the figure. Find the smallest value of R which permits all the
light to pass around the arc.

Observer

10. A glass sphere has a radius of 5.0 cm and a refractive index of


1.6. A paperweight is constructed by slicing through the sphere
on a plane that is 2.0 cm from the centre of the sphere and
perpendicular to a radius of the sphere that passes through the
center of the circle formed by the intersection of the plane and
the sphere. The paperweight is placed on a table and viewed
directly above by an observer who is 8.0 cm from the table
top as shown in figure. when viewed through the paperweight,
how far away does the tabletop appear to the observer?

8.0 cm
3.0 cm
5.
0

cm

11. A ray of light is incident on a composite slab at a angle of


incidence i as shown in the figure. Find the lateral shift x
of the ray when it comes out from the otherside.

12. A prism of apex angle A is made up of a material of refractive


index . The refractive indices of the mediums on the left and
right sides are 1 and 2 respectively. A ray of light is incident
from the side of medium of refractive index 1 at an angle i and
comes out from the other side as shown in the figure. Find the
angle of deviation.

www.physicsashok.in

35

OPTICS
13. A hemisphere of radius a/2 and made up of a material of variable refractive
index is placed with its base centre O at the origin as shown in the figure.
a
.
ax
A ray of light is incident at the point O at an angle with the normal in
the xy plane and comes out through a point P on its curved surface.
Find the coordinate of the point P if 0 .

The refractive index of the material of the hemisphere varies as

14. A ray of light is incident on the sphere of radius R and refractive index as
shown in the figure. The incident ray is parallel to a horizontal diameter and
the distance between the incident ray and the horizontal diameter is b. Find
the angle of deviation suffered by the ray..

b
R

15. An intense beam parallel to the principal axis is incident on


a convex lens. Multiple extra images F 1, F 2, ...... are formed Principal
due to feeble internal reflections, called flare spots as shown
axis
F1
in the figure. The radius of curvature of the lens is 30 cm and
60 cm and the refractive index is 1.5.
Find the position of the first flare spot.

F2 F0

16. The image of the object shown in the figure is formed at the bottom of the tray filled with water. From the
details given in the figure, calculate the value of h.
= 30 cm
/4
O

36 cm

1m

85 cm

17. In the given figure there are two thin lenses of same focal
length arranged with their principal axes inclined at an
angle . The separation between the optical centers of the
lenses is 2 . A point object lies on the principal axis of the
O
convex lens at a large distance to the left of convex lens.
(a) Find the coordinates of the final image formed by the
system of lenses taking O as the origin of coordinate axes, and
(B) Draw the ray diagram.

www.physicsashok.in

36

OPTICS
18. (a) A prism has refracting angle equal to /2. It is given that is the angle of minimum deviation and
is the deviation of the ray entering at grazing incidence. Prove that sin = sin2 and cos = cos
(b) A ray of light passes through a prism in a principal plane the deviation being equal to angle of
2
incidence which is equal to 2 . It is given that is the angle of prism. Show that cos

8 2 1

where is the refractive index of the material of prism.


19. A thin flat glass plate is placed in front of a convex mirror.
At what distance b from the plate should a point source of
light S be placed so that its image produced by the rays
reflected from the front surface of the plate coincides with
the image formed by the rays reflected from the mirror?
The focal length of the mirror is = 20 cm and the
distance from the plate to the mirror a = 5 cm. How can
the coincidence of the images be established by direct
observation?

20. A concave mirror forms the real image of a point source lying on the
optical axis at a distance of 50 cm from the mirror. The focal length
of the mirror is 25 cm. The mirror is cut in two and its halves are drawn
a distance of 1 cm apart in a direction perpendicular to the optical axis.
How will the images formed by the halves of the mirror be arranged?

1 cm

21. A glass hemisphere of radius 10 cm and = 1.5 is silvered over tis curved surface. There is an air
bubble in the glass 5 cms from the plane surface along the axis. Find the position of the images of this
bubble seen by observer looking along the axis into the flat surface of the hemisphere.
22. The height of a candle flame is 5 cm. A lens produces an image of this flame 15 cm high on a screen.
Without touching the lens, the candle is moved over a distance of = 1.5 cm away from the lens, and a
sharp image of the flame 10 cm high is obtained again after shifting the screen. Determine the main
focal length of the lens.
23. A thin converging lens of focal length is moved between a candle and a screen. The distance between
the candle and the screen is d (> 4 ). Show that for two different positions of the lens, two different
images can be obtained on the screen. If the ratio of dimensions of the image is , find the value of (
+ 1/ ).
24. Three convergent thin lenses of focal lengths 4a, a and 4a respectively are placed in order along the
axis so that the distance between consecutive lenses is 4a. Prove that this combination simply inverts
every small object on the axis without change of magnitude or position.
25. A converging bundle of light rays in the shape in the shape of a cone with the vertex angle of 40 falls on
a circular diaphragm of 20 cm diameter. A lens with a focal power of 5 diopters is fixed in the diaphragm.
What will the new cone angle be?
26. A ray of light is incident on the spherical surface of radius of curvature R
as shown in the figure. Therefractive index on the right side of spherical
surface is . The medium ont he left side of the spherical surface is air..
The distance of the incident ray from the axis of the spherical surface is
b. After refraction the ray intersects the axis at a point. F. Find the
distance of the point F from the pole O.

www.physicsashok.in

37

OPTICS
10 cm
27. Consider an arrangement of two equibi convex lenses of focal length
in air 10 cm. The refractive index of the glass of which the lenses are
g
between the made is g = 3/2 and the refractive index of water filling
the space two lenses is w = 4/3. A small object O is placed on the axis O AIR
at a distance of 10 cm from the first lens in air as shown in the figure. The
distance of separation between the two lenses is 10 cm. Find the
position and magnification of the final image.

10 cm
g
AIR

28. A thin convex lens of focal length 1m is cut into three parts A, B and
C along the diameter. The thickness of the middle layer C is 1 cm. The
middle layer is now removed and the two parts A nad B are put together
to form a composite lens. Then the part C is also placed infront of this
composite lens symmetrically as shown in the figure. A paraxial beam
of light is incident along tyhe axis of the part C. Find the distance
between the two images formed.

1 cm

B
A

B
29. An equi biconvex lens of focal length 10 cm in AIR and made up
of material of refractive index 3/2 is polished on one side. Another
identical lens (not polished) is placed infront of the polished lens
at a distance of 10 cm as shown in the figure. The space between
the two lenses is filled with a liquid of refractive index 4/3. An object
O is placed infront of the unpolished lens at a distance of 10 cm.
Find the final position of the image.

10 cm

10 cm

B
60
30. Consider an equilateral prism ABC as shown in the figure. A ray
of light is incident on the face AB and gets transmitted into the
prism. Then total internal reflection takes place at the face BC
and the ray comes out of prism through the face AC. The total
angle of deviation is 120. Find the refractive index of the
material of the prism.

60
A

60
C

Level # 3
1.

An object is placed 21 cm in front of a concave mirror of radius of curvature 10 cm. A glass slab of
thickness 3 cm and refractive index 1.5 is then placed close to the mirror in the space between the
object and the mirror.
Find the position of the final image formed.
(You may take the distance of the near surface of the slab from the mirror to be 1 cm). [IIT 1980]

2.

The x-y plane is the boundary between two transparent media. Medium 1 with z 0 has refractive
index

2 and medium 2 with z 0. 0, has a refractive index 3 . A ray of light in medium 1 given by

the vector A 6 3 i 8 3 j 10 k is incident on the plane of separation. Find the unit vector in the direction
of the refracted ray in medium 2.
[IIT 1999]

3.

An object is placed in front of a convex mirror at a distance of 50 cm. A plane mirror is introduced
covering lower half of the convex mirror. If the distance between the object and the plane mirror is 30
cm, it is found that there is no parallux between the images formed by two mirrors. What is the radius of
curvature of the convex mirror?
[IIT 1973]

www.physicsashok.in

38

OPTICS
4.

A rectangular block of glass is placed on a printed page lying on a horizontal surface. Find the value of
the refractive index of glass for which the letters on the page are not visible from any of the vertical faces
of the block.
[IIT 1979]

5.

A glass lens has focal length 5 cm in air. What will be its focal length in water. (Refractive index of glass
is 1.51 and that of water is 1.33).
[IIT 1977]

6.

A ray of light is travelling form diamond to glass. Calculate the minimum angle of incidence of the ray as
the diamond glass interface such that no light is refracted into glass. W hat will happen if the angle of
incidence exceeds the angle? (refractive index of glass is 1.51 and that of diamond is 2.47)[IIT 1977]

7.
8.

What is the velocity of light in glass of refractive index 1.5? (Velocity of light in air = 3 x 1010 cm/sec.)
[IIT 1976]
Photographs of the ground are taken from an aircraft flying at an altitude of 2000 meters by a camera
with a lens of focal length 50 cm. the size of the film in the camera is 18 cm x 18 cm. What area of the
ground can be photographed by this camera at any one time?
[IIT 1976]

9.

A rectangular glass block of thickens 10 cm and refractive index 1.5 is placed over a small coin. A
4
to a height of 10 cm and is placed over the glass block.
3
(a) Find the apparent position of the object when it is viewed at near normal incidence.
(b) Draw a neat ray diagram.
(c) If the eye is slowly moved away from the normal at a certain position the object is found to disappear
due to total internal reflection. At which surface does this happen and why?
[IIT 1975]

beaker filled with water of refractive index

10. A ray of light travelling in air is incident at grazing angle


(incident angle = 90) on a long rectangular slab of a
transparent medium of thickness t = 1.0 m (see figure).
The point of incidence is the origin A (0, 0). The medium
has a variable index of refraction n (y) given by
n(y) = [Ky3/2 + 1]
3/2
where K = 1.0 (meter)
The refractive index of air is 1.0.
(a) Obtain a relation between the slope of the trajectory of
the ray at a point B (x, y) in the medium and the incident
angle at that point.
(b) Obtain an equation for the trajectory y (x) of the ray
in the medium.
(c) Determine the coordinates (x 1, y1) of the point P, where the ray intersects the upper surface of the
slab-air boundary.
(d) Indicate the path of the ray subsequently.
[IIT 1995]

11. A quarter cylinder of radius R and refractive index 1.5 is


placed on a table. A point object P is kept at a distance
of mR from it. Find the value of m for which a ray from P
will emerge parallel to the table as shown in the figure.
[IIT 1999]

12. A light ray is incident on an irregular shaped slab of refractive index


2 at an angle of 45 with the normal on the incline face as shown
in the figure. the ray finally emerges from the curved surface in the
medium of the refractive index = 1.514 and passes through point E.
If the radius of curved surface is equal to 0.4 m, find the distance OE
correct up to two decimal places. [IIT 2004]

www.physicsashok.in

39

OPTICS
13. A point object O is placed at a distance of 12 cm on the axis of a convex lens of focal length 10 cm. On the
other side of the lens, a convex mirror is placed at a distance of 10 cm from the lens such that the image
formed by the combination coincides with the object itself. What is the focal length of the convex mirror?
[IIT 1976]
14. An object of height 4 cm is kept to the left of and on
the axis of a converging lens of focal length 10 cm as
shown in figure. A plane mirror is placed inclined at 45
to the lens axis 10 cm to the right of the lens (see
figure). Find the position and size of the image
formed by the lens and mirror combination.
trace the rays forming the image. [IIT 1972]

15. An object is placed at 20 cm left of the convex lens of focal


length 10 cm. If a concave mirror of focal length 5 cm is placed
at 30 cm to the right of the lens find the magnification and the
nature of the final image. Draw the ray diagram and locate the
position of the final image.
[IIT 1974]

16. An object is approaching at thin convex lens of focal length 0.3 m


with a speed of 0.01 m/s. Find the magnitudes of the rates of change
of position and lateral magnification of image when the object is at a
distance of 0.4 m from the lens.
[IIT 2004]

3
is placed on a horizontal
2
plane mirror as shown in the figure. The space between the lens

17. A thin biconvex lens of refractive index

4
.
3
It is found that when a point object is placed 15 cm above the
lens on its principle axis, the object coincides with its own image.
On repeating with another liquid, the object and the image again coincide at a distance 25 cm from the
lens. Calculate the refractive index of the liquid.
[IIT 2001]

and the mirror is then filled with water of refractive index

18. A convex lens of focal length 15 cm and a concave mirror of focal length
30 cm are kept with their optic axes PQ and RS parallel but separated
in vertical direction by 0.6 cm as shown. The distance between the lens
and mirror is 30 cm. An upright object AB of height 1.2 cm is placed
on the optic axis PQ of the lens at a distance of 20 cm from the lens.
If AB is the image after refraction from the lens and reflection from the
mirror, find the distance of AB from the pole of the mirror and obtain
its magnification. Also locate position of A and B with respect to the
optic axis RS.
[IIT 2000]

www.physicsashok.in

40

OPTICS
3
and
2
of focal length 0.3 m in air is sealed into an opening at one

19. A thin equiconvex lens of glass of refractive index

end of a tank filled with water . On the opposite side


3

of the lens, a mirror is placed inside the tank on the tank wall
perpendicular to the lens axis, as shown in figure. The separation
between the lens and the mirror is 0.8 m. A small object is placed outside the tank in front of the lens at
a distance of 0.9 m from the lens along its axis. Find the position (relative to the lens) of the image of the
object formed by the system.
[IIT 1997, May]

20. A thin plano-convex lens of focal length is split in to two halves:


one of the halves is shifted along the optical axis (see figure).
The separation between object and image planes is 1.8 m. The
magnification of the image formed by one of the half-lenses is 2.
Find the focal-length of the lens and separation between the two
halves. Draw the ray diagram for image formation.
[IIT 1996]
21. A plano convex lens has a thickness of 4 cm. When placed on a horizontal table with the curved surface
in contact with it, the apparent depth of the bottom most point of the lens is found to be 3 cm. If the lens
is inverted such that the plane face is in contact with the table, the apparent depth of the centre of the
plane face is found to be 25/8 cm. Find the focal length of the lens.
[IIT 1984]
22. The convex surface of a thin concavo-convex lens of glass of
refractive index 1.5 has a radius of curvature 20 cm. The concave
surface has a radius of curvature 60 cm. The convex side is
silvered and placed on a horizontal surface.
(a) Where should a pin be placed on the optic axis such that
its image is formed at the same place?
4
(b) If the concave part is filled with water of refractive index ,
3
find the distance through which the pin should be moved so
that the image of the pin again coincide with the pin.
[IIT 1981]

23. Find the focal length of the lens shown in the figure. The
radii of curvature of both the surfaces are equal to R.
[IIT 2003]

24. The refractive indices of the crown glass for blue and red lights are 1.51 and 1.49 respectively and those
of the flint glass are 1.77 and 1.73 respectively. An isosceles prism of angle 6 is made of crown glass.
A beam of white light is incident at a small angle of this prism. The other flint glass isosceles prism is
combined with the crown glass prism such that there is no deviation of the incident light. Determine the
angle of the flint glass prism. Calculate the net dispersion of the combined system.
[IIT 2001]

25. A prism of refracting angle 30 is coated with a thin film of transparent material of
is
refractive index 2.2 on face AC of the prism. A light of wavelength 5500 A
incident on face AB such that angle of incidence is 60, find
(a) the angle of emergence,
[Given refractive index of the material of the prism is

3 ].

www.physicsashok.in

41

OPTICS
(b) the minimum value of thickness of the coated film on the face AC for which the light emerging
from the face has maximum intensity.
[IIT 2003]
26. A right angle prism (45 90 45) of refractive index n has a plate of refractive
index n1 (n1 < n) cemented to its diagonal face. The assembly is in air. A ray is
incident on AB (see figure),
(a) Calculate the angle of incidence at AB for which the ray strikes the
diagonal face at the critical angle.
(b) Assuming n = 1.352, calculate the angle of incidence at AB for which
the refracted ray passes through the diagonal face undeviated.
[IIT 1996]
27. A right angled prism is to be made by selecting a proper material
and the angles A and B (B A), as shown in figure. It is desired
that a ray of light incident on the face AB emerges parallel to the
incident direction after two internal reflections.
(a) What should be the minimum refractive index n for this
to be possible?
(b)

5
is it possible to achieve this with the angle B
3
equal to 30 degrees ?
[IIT 1987]

For n =

28. Monochromatic light is incident on a plane interface AB


between two media of refractive indices n1 and n2 (n2>n1)
at an angle of incidence as shown in the figure. The
angle is infinitesimally greater than the critical angle
for the two media so that total internal reflection takes place.
Now if a transparent slab DEFG of uniform thickness and of
refractive index n3 is introduced on the interface (as shown in
the figure), show that for any value of n3 all light will ultimately
be reflected back again into medium II. Consider separately the cases.
(i) n3 < n1
and
(ii)
n3 > n1.
[IIT 1986]
4
) is refracted by a spherical air bubble of
3
radius 2 mm situated in water. Assuming the light rays to be paraxial,
(a) find the position of the image due to refraction at the first surface and the position of the final
image.
(b) draw a ray diagram showing the positions of both the images.
[IIT 1988]

29. A parallel beam of light travelling in water (refractive index =

30. Light is incident at an angle on one planar end of a transparent cylindrical rod of refractive index n.
Determine the least value of n so that the light entering the rod does not emerge from the curved surface
of the rod irrespective of the value of .
[IIT 1992]
31. The radius of curvature of the convex face of a plano convex lens is 12 cm and its refractive index is 1.5.
(i) Find the focal length of this lens.
(ii) The plane surface of the lens is now silvered. At what distance from the lens will parallel rays
incident on the convex face converge.
(iii) Sketch the ray diagram to locate the image, when a point object is placed on the axis, 20 cm from
the lens (polished).
(iv) Calculate the image distance when the object is placed as in (iiii).
[IIT 1979]
32. A ray of light is incident at an angle of 60 on one face of prism which has an angle of 30 with the
incident ray. Show that the emergent ray is perpendicular to the face through which it emerges and
calculate the refractive index of the material of the prism.
[IIT 1978]

www.physicsashok.in

42

OPTICS
33. A pin is placed 10 cm in front of a convex lens of focal length 20 cm., made of a material of refractive index
1.5. The surface of the lens farther away from the pin is silvered and has a radius of curvature are 22 cm.
Determine the position of the final image. Is the image real as virtual?
[IIT 1978]
34. The refractive index of the material of a prism of refracting angle 45 is 1.6 for a certain monochromatic
ray. What should be minimum angle of incidence of this ray on the prism so that no total internal reflection
takes place as the ray comes out of the prism.
[IIT
1976]
35. A prism of refractive index n1 and another prism of refractive index n2
are stuck together without a gap as shown in the figure. The angles
of the prisms are as shown n1 and n2 depend on , the wavelength
of light, according to
n1 1.20

10.8 10 4
2

and

n2 1.45

1.80 10 4
2

where, is in nm.
(a) Calculate the wavelength 0 for which rays incident at any angle on the interface BC pass through
without bending at that interface.
(b) For light of wavelength 0, find the angle of incidence i on the face AC such that the deviation
produced by the combination of prisms is minimum.
[IIT 1998]
36. A projector lens has a focal length 10 cm. It throws an image of a 2 cm x 1 cm slide on a screen 5 metre
from the lens. Find :
(a) the size of the picture on the screen and
(b) ratio of illuminations of the slide and of the picture on the screen.
[IIT 1975]
37. A ray of light incident normally on one of the faces of a right angled
isosceles prism is found to be totally reflected as shown in the figure.
What is the minimum value of the refractive index of the material of
the prism? When the prism is immersed in water, trace the path of
the emergent rays for the same incident ray, indicating the values
4

of all the angles. .


3

[IIT 1973]

www.physicsashok.in

43

OPTICS

Answer Key
Assertion & Reasion

Assertion & Reasion


Que.

10

Ans.

Que.

11

12

13

14

15

16

Ans.

Level # 1

Objective Type
Que.

10

Ans.

Que.

11

12

13

14

15

16

17

18

19

20

Ans.

CD

Que.

21

22

23

24

25

26

27

28

29

30

Ans.

Que.

31

32

33

34

35

36

37

38

39

40

Ans.

Que.

41

42

43

44

45

46

47

48

49

50

Ans.

Que.

51

52

53

54

55

56

57

58

59

60

Ans.

BD

Que.

61

62

63

64

Ans.

BC

CD

Fill in the Blanks / TrueFalse / Match Table


1. 2 x 108 m/s, 0.4 x 106 m

2. d = +15 cm

5. 60 cm

6.

9. Zero
15

10. 5 x 1014 Hz, 4000

25
9

3. 4000, 5 x 1014 Hz

4. 2

7. 35 cm

8. 1.3

11. 0.125 m, 0.5 m

1 2 .

Que.

13

14

15

16

17

18

19

20

Ans.

www.physicsashok.in

44

OPTICS

Passage Type
Que.

10

11

12

13

14

15

Ans.

Que.

16

17

18

19

20

21

22

23

24

25

26

27

28

29

30

Ans.

Que.

31

32

33

34

35

36

37

38

39

40

41

42

43

44

45

Ans.

Que.

46

47

48

49

50

51

52

53

54

55

56

57

58

59

60

Ans.

Que.

61

62

63

64

65

66

67

68

69

70

71

Ans.

Level # 2.
2. 1.6

7.

3. 90 cm., Yes.

4. 2.88 m

1 2 d 2 1
x

r
0
0
(a) A

(c) Ray will become parallel to y-axis.

8. 2 Km.

9. R 12 cm .

1 1
12. i A sin
2

10. 7.42 cm

sin 2 i cos A sin i


sin A
1

(b)

5. (a) 13.3 cm (b) 14.975 cm


0

1
2 2


2
2
0 0 1
r


cos i
t 1
x

1
11.

12 sin 2

13.

cos i

1
sin i

t
2

2
2
i
sin i

a a

15
,
8 8

b2
b2
1 b
2

2
sin

14.
15. The first spot is at 12 cm on left side from the optical
R
R
R2

centre.

2 cos 1
, 0
16. 20 cm
17.
19. b = 15 cm.
cos 1

20. At a distance of 15 cm. from the mirror 2 cm. from each other.

21. First Image at a distance of 3.33 cm from flat surface and the second at infinity.
22. 9 cm

1
2

R 1
d

2
2
23. 2 2
25. 8140
26.
b
b

2 2 1 2

R
R
27. 25 cm from the second lens on the right side magnification m = 2.
28. 0.5 cm
29. 6 cm back side of unpolished lens.

30.

www.physicsashok.in

7
3

45

OPTICS

Level # 3
3 i 4 j 5 k

1. 7.67 cm from the mirror.

2.

5. 18.84 cm

6. = sin1 0.6115 = 38 No ray is refracted into glass.

7. 2 x 1010 cm per second.

8. 720 m x 720 m

9. (a) 14.16 cm below the water surface.


10. (a) Slop

dy
cot i
dx

2 x
(b) y K
4

3. 25 cm

5 2

4.

(b) No glass water interface.


4

(c) (4m, 1m)

(d) Ray emerges parallel to the positive x-axis.

4
12. OE = 6.06 m
13. 25 cm.
3
14. At a perpendicular distance of 20 cms from the lens axis 8 cm is size oriented parallel to lens axis.

11. m

15. At the position of the object magnification = 1.

16. 0.09 m/s, 0.3 per second.

17. 1.6

18. Distance of AB from pole of mirror a15 cm, magnification = 1.5.


Distance of A above RS is 0.3 cm,
Distance of B below RS is 1.5 cm.
19. 0.9 m from the lens (0.1 m behind the mirror)

20. (a) = 0.4 m, d = 0.6 m.

21. 75 cm

22. (a) 15 cm

(b) 1.15 cm towards the lens

3 R
23.
3
1

24. 4, 0.04

25. (a) 0

(b) 1250

1
27. (a) sin B

(b) No.

1 1
2
2

n n1 n1
26. (a) sin

1
(b) sin

1.352

73

29. (a) Image due to first surface at a distance of 6 mm before the first surface final image at a distance of
1mm before the first surface.
30.

31. (a) 24 cm

33. 17 cm infront of lens, Real.


36. (a) 100 cm x 50 cm

(b) 12 cm

(d) 80 cm.

34. sin 1 0.176 10.1

slide
2401
(b)
picture

32. 3 .
35. (a) 0 600 nm

(b) i sin1

37. 2 , Angle of refraction in water r = sin 1 .

XXXX

www.physicsashok.in

46

You might also like